LSAT and Law School Admissions Forum

Get expert LSAT preparation and law school admissions advice from PowerScore Test Preparation.

 Administrator
PowerScore Staff
  • PowerScore Staff
  • Posts: 8929
  • Joined: Feb 02, 2011
|
#37056
Please post below with any questions!
 mcassidy1
  • Posts: 8
  • Joined: Sep 03, 2019
|
#67829
Why is answer b correct. I took the passage to mention companies in general not a specific company.
 Rachael Wilkenfeld
PowerScore Staff
  • PowerScore Staff
  • Posts: 1392
  • Joined: Dec 15, 2011
|
#67838
Hi mcassidy1,

This is a whole to parts flaw. Just because they are advocating something for the economy as a whole doesn't mean that they are advocating it for each of the component parts. If you look closely at the phrasing of the stimulus, it does not talk about "companies" in the aggregate, but a single company increasing productivity by firing workers. Because of this, we can see the error in going from the whole of the economy to a single company.

Hope that helps!
Rachael
User avatar
 noelleja
  • Posts: 5
  • Joined: Jun 24, 2023
|
#102501
During the exam, I was between A & B, and I ultimately ended up incorrectly selecting A. In retrospect, I see the error of composition much more clearly (i.e., the author attempts to make what is true of a part--one company--true of the whole--the economy at large). At the same time though, I still feel that A may be a source of vulnerability for the argument since it is true that the author fails to explain that unemployment alone is enough to severely dampen the plan for increased productivity. Why can't politicians proceed with their plan even if unemployment is one of the consequences of their plan? What if the benefits far outweigh the risk (i.e., increased unemployment)?

When reading the choice initially, I didn't see "sufficient" as describing a conditional error since the stimulus didn't contain a conditional statement, but is it possible that the one term in the answer choice was enough to allude to a conditional error? Or is this simply an instance where a tester is forced to select the best answer? And, if that's the case, is the better answer almost always going to be easily identifiable as one of the common flaws described in the LR Bible?

Thanks in advance,
Noelle
User avatar
 Jeff Wren
PowerScore Staff
  • PowerScore Staff
  • Posts: 451
  • Joined: Oct 19, 2022
|
#102591
Hi Noelle,

Whenever you're considering an answer for a flaw question, it's a good idea to immediately compare the answer under consideration to the argument above (specifically the conclusion) to see whether the argument actually commits this flaw. The reason that this is so important is that many tempting wrong answers describe flaws that would be true of very similar arguments, but not exactly the argument given. (Think of these as "shell game" wrong answers, just like there are "shell game" wrong answers in other questions.)

In Answer A, for example, does the argument actually presume that increased unemployment is sufficient to abandon increased productivity as an economic goal? Does the argument even recommend abandoning the goal of increased productivity?

It may seem like that is where the argument is heading, but nowhere in the argument itself is such a recommendation actually made. The conclusion of the argument is simply that attempting to increase productivity of the whole economy may benefit business owners but will also increase the number of unemployed workers. The implications of that conclusion and any policy recommendations based on it are left unanswered.

If the argument had then gone on to conclude, "therefore we must abandon this policy," then Answer A would correctly describe a flaw that had occurred. The argument as it is written is simply bringing up a negative/drawback of the policy without going so far as to fully weigh the pros and cons.

So the word "sufficient" often is used when describing a conditional error, but it would be a mistake to rule out an answer simply because of this word. As described above, it the argument had gone further in concluding that the policy should be abandoned (presumably because the unemployment increase outweighs the productivity increase), then Answer A would have been correct even though this isn't a conditional argument. However, if an answer had stated "the argument mistakes a sufficient condition for a necessary condition," that answer should be easy to eliminate since this argument isn't conditional.

Unfortunately, the best answer is not always going to be one of the easily identifiable flaws described in "The Logical Reasoning Bible." That would be too easy! While knowing those flaws is definitely helpful, and they will be correct in some questions (like this one), the test makers have recently been using more flaws that don't nicely fit into traditional classification in order to make these questions more difficult.

Get the most out of your LSAT Prep Plus subscription.

Analyze and track your performance with our Testing and Analytics Package.